Đến nội dung

xuanhoan23112002 nội dung

Có 95 mục bởi xuanhoan23112002 (Tìm giới hạn từ 05-05-2020)



Sắp theo                Sắp xếp  

#669654 MỘT SỐ PHƯƠNG PHÁP GIẢI TOÁN TỔ HỢP THCS

Đã gửi bởi xuanhoan23112002 on 24-01-2017 - 06:11 trong Toán rời rạc

Trên mặt phẳng cho 5 điểm có tọa độ nguyên trong đó không có 3 điểm nào thẳng hàng .Chứng minh rằng trong số các tam giác tạo thành từ 5 điểm đã cho có ít     nhất 3 tam giác có diện tích nguyên




#668399 MỘT SỐ PHƯƠNG PHÁP GIẢI TOÁN TỔ HỢP THCS

Đã gửi bởi xuanhoan23112002 on 15-01-2017 - 12:32 trong Toán rời rạc

ko vi no ch

 

Bài 15 : Một bạn cờ quốc tế $8\times 8$  . Hỏi rằng quân mã có thể đi nước đầu tiên từ ô dưới cùng bên trái và kết thúc ở ô trên cùng bên phải không ? Với điều kiện nó phải đi qua tất cả các ô trên bàn cờ và mỗi ô chỉ đi qua đúng một lần. 

ko vi quan ma ko the di het ban co




#662739 Chuyên đề : Làm mạnh BĐT CôSy

Đã gửi bởi xuanhoan23112002 on 22-11-2016 - 20:51 trong Bất đẳng thức và cực trị

Tìm giá trị nhỏ nhất của 

P=$\fn_jvn \frac{a}{bc}$+$\fn_jvn \frac{2b}{ca}$+$\fn_jvn \frac{5c}{ab}$

trong đó a2+b2+c2=6




#706068 Topic ôn thi hình học vào cấp 3 chuyên

Đã gửi bởi xuanhoan23112002 on 16-04-2018 - 21:14 trong Hình học

Bài 19 (Đề thi vào lớp 10 chuyên Nam Định 2017):Cho tam giác ABC có ba góc nhọn nội tiếp đường tròn (O), AB < AC. Các tiếp tuyến của đường tròn (O) tại B và C cắt nhau tại M. Đường thẳng qua M song song với AB cắt đường tròn (O) tại D và E (D thuộc cung nhỏ BC), cắt BC tại F, cắt AC tại I.

1) Chứng minh năm điểm M, B, O, I, C cùng thuộc một đường tròn

2)Chứng minh$\frac{FI}{FE}=\frac{FD}{FM}$

3) OI cắt (O) tại P và Q (P thuộc cung nhỏ AB). QF cắt (O) tại T (T khác Q). Tính tỉ số $\frac{TQ^2+TM^2}{MQ^2}$




#642083 $\boxed{\text{Chuyên Đề}}$ Phương trình vô tỉ - Hệ phương...

Đã gửi bởi xuanhoan23112002 on 25-06-2016 - 09:09 trong Phương trình, hệ phương trình và bất phương trình

giúp mình bài này với: $x^4+1=5x(x^2-2)$




#662621 $\boxed{\text{Chuyên Đề}}$ Phương trình vô tỉ - Hệ phương...

Đã gửi bởi xuanhoan23112002 on 21-11-2016 - 17:24 trong Phương trình, hệ phương trình và bất phương trình

Các bạn suy nghĩ bài này xem:

giải hệ phương trình:

     5x2+ 2y2+ z=2

     xy+ yz+ xz =1




#668372 $\boxed{\text{Chuyên Đề}}$ Phương trình vô tỉ - Hệ phương...

Đã gửi bởi xuanhoan23112002 on 15-01-2017 - 10:34 trong Phương trình, hệ phương trình và bất phương trình

Giải phương trình này: $\sqrt{1-\frac{1}{x}}+\sqrt{x^2-x}=2$




#662648 $\boxed{\text{Chuyên Đề}}$ Phương trình vô tỉ - Hệ phương...

Đã gửi bởi xuanhoan23112002 on 21-11-2016 - 20:49 trong Phương trình, hệ phương trình và bất phương trình

Các bài làm hầu như không ghi ĐKXĐ vào, các bạn chú ý nhé. (Nhớ trích dẫn đề nữa)

 

Giải pt:

19) $\sqrt{2x^2+8x+6}+\sqrt{x^2-1}=2x+2$

 

20) $\sqrt[3]{24+x}+\sqrt{12-x}=6$

 

21) $(x+1)(x+4)=5\sqrt{x^2+5x+28}$

 

22) $4\sqrt{(4-x)(2+x)}=x^2-2x-12$

 

23) $\sqrt{3x-2}+\sqrt{x-1}=4x-9+2\sqrt{3x^2-5x+2}$

 

24) $5\sqrt{x}+\frac{5}{2\sqrt{x}}=2x+\frac{1}{2x}+4$

 

25) $x^2-2x=2\sqrt{2x-1}$

 

26) $x^3+1=2\sqrt[3]{2x-1}$

câu 21:DKXD: với mọi x

sau đó đặt cận vệ phải là a(a>=0) tôi biểu diễn về trại theo a và phân tích thành tích là xong




#662624 $\boxed{\text{Chuyên Đề}}$ Phương trình vô tỉ - Hệ phương...

Đã gửi bởi xuanhoan23112002 on 21-11-2016 - 17:36 trong Phương trình, hệ phương trình và bất phương trình

Giải phương trình  x2 - 5x + 36 = 8 $\sqrt{3x+4}$

DKXD:x>=-4/3

Cậu bình phương cả 2 vế lên:

(x2-5x+36)2=64(3x+4)

x4-10x3+ 97x- 552x+1040=0

(x-4)2(x- 2x + 65)=0

x=4 (thỏa man DKXD)




#704835 TOPIC thảo luận, trao đổi toán thi học sinh giỏi khối 10,11 .

Đã gửi bởi xuanhoan23112002 on 03-04-2018 - 22:54 trong Chuyên đề toán THPT

Đóng góp bài này cho topic

Cho a,b,c,d  thỏa mãn a+b+c+d=6 và a2+b2+c2+d2=12

cmr 48 >= 4(a3+b3+c3+d3) -(a4+b4+c4+d4) >= 36

Lời giải:
4(a3+b3+c3+d3) - (a4+b4+c4+d4) = -((a-1)4+(b-1)4+(c-1)4+(d-1)4)+52
Đặt x=a-1, y=b-1, z=c-1, t=d-1.
Bất đẳng thức cần chứng minh tương đương với:
 16>= x4+y4+z4+t4>=4
Ta cũng có: x2+y2+z2+t2=4
Từ đây áp dụng bất đẳng thức AM-GM ta có điều phải chứng minh(Q.E.D) 
Đẳng thức xảy ra tại (a,b,c,d) = (3,1,1,1) và (2,2,2,0) và các hoán vị của nó



#706221 Dạng toán: Trò chơi

Đã gửi bởi xuanhoan23112002 on 17-04-2018 - 21:40 trong IQ và Toán thông minh

max khó




#706709 [TOPIC] ÔN THI BẤT ĐẲNG THỨC $\boxed{\text{THPT CHUYÊN}}$...

Đã gửi bởi xuanhoan23112002 on 22-04-2018 - 23:11 trong Tài liệu - Đề thi

Cách của bạn Linh đúng rồi mọi người thử tìm các cách khác chẳng hạn như dùng nguyên lí Đirichlet




#706706 [TOPIC] ÔN THI BẤT ĐẲNG THỨC $\boxed{\text{THPT CHUYÊN}}$...

Đã gửi bởi xuanhoan23112002 on 22-04-2018 - 23:02 trong Tài liệu - Đề thi

Bài 78(IMO 1984): Cho a, b, c là các số không âm thỏa mãn $a+b+c=1$.CMR:

$0\leq ab+bc+ca-2abc\leq \frac{7}{27}$




#706710 [TOPIC] ÔN THI BẤT ĐẲNG THỨC $\boxed{\text{THPT CHUYÊN}}$...

Đã gửi bởi xuanhoan23112002 on 22-04-2018 - 23:14 trong Tài liệu - Đề thi

Bài 79: Cho a, b, c là các số thực dương thỏa mãn $a+b+c=3$. CMR:

$(a^2-a+1)(b^2-b+1)(c^2-c+1)\geq 1$




#706768 [TOPIC] ÔN THI BẤT ĐẲNG THỨC $\boxed{\text{THPT CHUYÊN}}$...

Đã gửi bởi xuanhoan23112002 on 23-04-2018 - 19:56 trong Tài liệu - Đề thi

Bài 85: Cho a, b, c là các số thực dương thỏa mãn $a^2+b^2+c^2+abc=4$. CMR: 

$a+b+c\geq \sqrt{a}+\sqrt{b}+\sqrt{c}$




#707240 [TOPIC] ÔN THI BẤT ĐẲNG THỨC $\boxed{\text{THPT CHUYÊN}}$...

Đã gửi bởi xuanhoan23112002 on 29-04-2018 - 07:55 trong Tài liệu - Đề thi

Bài 106Cho a1, a2,...,a19 là các số tự nhiên thỏa mãn: a1+a2+...+a19 =26. Tìm giá trị nhỏ nhất của:

S=a12+a22+...+a192




#706705 [TOPIC] ÔN THI BẤT ĐẲNG THỨC $\boxed{\text{THPT CHUYÊN}}$...

Đã gửi bởi xuanhoan23112002 on 22-04-2018 - 22:59 trong Tài liệu - Đề thi

Bài 77(APMO 2004): Cho a, b, c là các số thực dương. CMR: $(a^2+2)(b^2+2)(c^2+2)\geq 9(ab+bc+ca)$




#709478 [TOPIC] ÔN THI BẤT ĐẲNG THỨC $\boxed{\text{THPT CHUYÊN}}$...

Đã gửi bởi xuanhoan23112002 on 29-05-2018 - 08:41 trong Tài liệu - Đề thi

Bài 138: Với a, b, c là các số thực dương. Tìm giá trị nhỏ nhất của biểu thức:

$P=\frac{2}{a+\sqrt{ab}+\sqrt[3]{abc}}-\frac{3}{\sqrt{a+b+c}}$




#706088 [TOPIC] ÔN THI BẤT ĐẲNG THỨC $\boxed{\text{THPT CHUYÊN}}$...

Đã gửi bởi xuanhoan23112002 on 16-04-2018 - 22:14 trong Tài liệu - Đề thi

Bài 13: Cho a, b, c >0 và a+b+c=1. CMR: $5(a^2+b^2+c^2)\leq 6(a^3+b^3+c^3)+1$

Đẳng thức xảy ra khi nào?




#706080 [TOPIC] ÔN THI BẤT ĐẲNG THỨC $\boxed{\text{THPT CHUYÊN}}$...

Đã gửi bởi xuanhoan23112002 on 16-04-2018 - 21:55 trong Tài liệu - Đề thi

Bài 10: Cho $a,b,c$ là các số thực dương thỏa mãn $abc=1$. Chứng minh rằng $\frac{\sqrt{a^{4}+b^{4}}}{1+ab}+\frac{\sqrt{b^{4}+c^{4}}}{1+bc}+\frac{\sqrt{c^{4}+a^{4}}}{1+ac}\geq 3$

Bất đẳng thức của bạn sai rồi vế phải là $\frac{3}{\sqrt{2}}$




#706058 [TOPIC] ÔN THI BẤT ĐẲNG THỨC $\boxed{\text{THPT CHUYÊN}}$...

Đã gửi bởi xuanhoan23112002 on 16-04-2018 - 20:21 trong Tài liệu - Đề thi

Bài 9: Đặt a+b+c=p,ab+bc+ca=q,abc=r

Theo giả thiết, $ =>p+r=4=>r=4=> p+r==> r=4p $ , từ giả thiết ta dễ dàng chứng minh được p$\geq$3

Theo bất đẳng thức Schur, ta có

$p^3-4pq + 9r \geq 0 => p^3-4pq + 9(4-p) \geq 0 =>  p^3- 9p+36 \geq 4pq => \frac{p^3-9p+36}{4p} \geq q$

$\Rightarrow \frac{p^3 -9p+36}{4p}\geq q$

Ta sẽ chứng minh p$\geq q$ hay ta chứng minh:p \geq \frac{p^3-9p+36}{4p}  <=> p^3 - 4p^2 -9p + 36 \leq 0 <=> 3 \leq p \leq 4(bất đẳng thức này hiển nhiên đúng)

Từ đó ta suy ra q$\leq$ 4

                     Vậy MaxP=4. Đẳng thức xảy ra chẳng hạn khi (a,b,c)=(0,2,2)
Bài 10: Cho a, b, c là các số thực không âm thỏa mãn $a^2+b^2+c^2+abc$=4
Tìm min và max của P=a+b+c



#706205 [TOPIC] ÔN THI BẤT ĐẲNG THỨC $\boxed{\text{THPT CHUYÊN}}$...

Đã gửi bởi xuanhoan23112002 on 17-04-2018 - 20:20 trong Tài liệu - Đề thi

Bài 25: Cho $0\leq a, b, c \leq 2$ và a+b+c=3

1. Tìm giá trị nhỏ nhất và giá trị lớn nhất của M =$a^2+b^2+c^2$

2. Tìm giá trị nhỏ nhất và giá trị lớn nhất của N =$a^3+b^3+c^3$

3. Tìm giá trị nhỏ nhất của H =$\sqrt{ab}+\sqrt{bc}+\sqrt{ca}$

P/s: Mỗi câu là 1 bài toán riêng mình ghép chung thành 1 bài




#706211 [TOPIC] ÔN THI BẤT ĐẲNG THỨC $\boxed{\text{THPT CHUYÊN}}$...

Đã gửi bởi xuanhoan23112002 on 17-04-2018 - 20:46 trong Tài liệu - Đề thi

Bài 26: Cho x, y, z là các số thực không âm thỏa mãn: $x+y+z=100$

Xác định giá trị lớn nhất của M =$11xy+3xz+2012yz$




#706223 [TOPIC] ÔN THI BẤT ĐẲNG THỨC $\boxed{\text{THPT CHUYÊN}}$...

Đã gửi bởi xuanhoan23112002 on 17-04-2018 - 21:53 trong Tài liệu - Đề thi

Bài 27: Cho a, b, c là các số không âm thỏa mãn: $a+b+c=3$

Chứng minh rằng $\sqrt{a}+\sqrt{b}+\sqrt{c}\geq ab+bc+ca$




#706686 [TOPIC] ÔN THI BẤT ĐẲNG THỨC $\boxed{\text{THPT CHUYÊN}}$...

Đã gửi bởi xuanhoan23112002 on 22-04-2018 - 21:11 trong Tài liệu - Đề thi

Bài 75: Cho a, b, c là các số thực dương. CMR:

$a^2+b^2+c^2+2abc+1\geq 2(ab+bc+ca)$